First Order PDE Cauchy problem Using Method of Characteristics

Click For Summary
The discussion revolves around solving a first-order PDE Cauchy problem using the method of characteristics. The characteristic equations derived are dx/x = dy/(y^2 + 1) = dU/(U - 1), leading to expressions for U in terms of x and y. The user struggles to determine the arbitrary function f after substituting Cauchy data into the equations. There is a mention of a relationship between the inverse tangent function and the natural logarithm, which may assist in finding f. The conversation highlights the complexities of reconciling the derived equations to find a solution for U.
pk415
Messages
4
Reaction score
0

Homework Statement


Ok, so I can get through most of this but I can't seem to get the last part... Here is the problem

xU_x + (y^2+1)U_y = U-1; U(x,x) = e^x


Homework Equations





The Attempt at a Solution



Characteristic equations are:

\frac{dx}{x} = \frac{dy}{y^2+1} = \frac{dU}{U-1}

Solving the first and third gives:

\frac{U-1}{x} = c_1

The first and second equation yield:

tan^{-1}(y) - lnx = c_2

Put the two together in the form

c_1 = f(c_2)

\frac{U-1}{x} = f(tan^{-1}(y) - lnx)

Sub in the Cauchy data and you get

\frac{e^x-1}{x} = f(tan^{-1}(x) - lnx)

Now how do I find what my arbitrary function f is? I have spent hours on this. Is there something that relates inverse tan to natural log? Arrggghhhh!

Thanks for any help.
 
Physics news on Phys.org
Where did the function f come from? The way I see it, you end up with three equations:

\begin{cases}<br /> x=C_1 e^{\tan^{-1}y}\\<br /> U=C_2 x+1\\<br /> U=C_3 e^{\tan^{-1}y}+1<br /> \end{cases}

Reconciling those, you get
U=Cxe^{\tan^{-1}y}+1



Oh, and there is a relation between inverse tan and natural log: http://functions.wolfram.com/ElementaryFunctions/ArcTan/02/0001/

\tan^{-1}z={i \over 2}\ln{1-iz \over 1+iz}
 
Question: A clock's minute hand has length 4 and its hour hand has length 3. What is the distance between the tips at the moment when it is increasing most rapidly?(Putnam Exam Question) Answer: Making assumption that both the hands moves at constant angular velocities, the answer is ## \sqrt{7} .## But don't you think this assumption is somewhat doubtful and wrong?

Similar threads

Replies
19
Views
2K
Replies
8
Views
1K
  • · Replies 1 ·
Replies
1
Views
974
Replies
5
Views
2K
  • · Replies 19 ·
Replies
19
Views
2K
  • · Replies 21 ·
Replies
21
Views
2K
  • · Replies 4 ·
Replies
4
Views
2K
  • · Replies 5 ·
Replies
5
Views
1K
  • · Replies 1 ·
Replies
1
Views
2K